Q21

 
cyruswhittaker
Thanks Received: 107
Forum Guests
 
Posts: 246
Joined: August 11th, 2010
 
 
trophy
Most Thanked
trophy
First Responder
 

PT 49, S 3, Q 21, P4 "Which one of the

by cyruswhittaker Sun Sep 26, 2010 1:18 pm

For this question, I was stuck between A and E, eventually choosing E.

I'm trying to show why A is the better answer, but I'm a little unclear. E doesn't appear right because of what it states: effective mechanism to prevent atmospheric gases, when only one gas, oxygen, is stated as bending competitively.

A, while correctly stating that the productivity is due to the C-4 process, only mentions maize, while in the first paragraph, and subsequent paragraphs, discuss the process in a more general context--for not just maize, but for these more productive plants in general.

So I guess I'm confused on A due to the scope. Both A and E seem to have a problem.
User avatar
 
bbirdwell
Thanks Received: 864
Atticus Finch
Atticus Finch
 
Posts: 803
Joined: April 16th, 2009
 
This post thanked 1 time.
 
 

Re: PT 49, S 3, Q 21, "Which one of the following..."

by bbirdwell Mon Sep 27, 2010 7:16 pm

A, while correctly stating that the productivity is due to the C-4 process, only mentions maize, while in the first paragraph, and subsequent paragraphs, discuss the process in a more general context--for not just maize, but for these more productive plants in general


I disagree. The mention of the few other plants is second to the passage's emphasis on maize. The entire first paragraph is about maize, and the stage for the entire explanation about C-4 is set with the statement: "biochemistry has revealed the mechanism underlying maize's impressive productivity."

(E) is wrong on several counts. First of all, only one mechanism is described and it is not described as "complex." You're right that the mechanism is mostly about oxygen and not "gases," but more to the point, oxygen is not "prevented from entering the leaves" -- it's just put in a different part of the leaf than the part that the rubisco is in.

You're doing your job if you get it down to these two choices.
I host free online workshop/Q&A sessions called Zen and the Art of LSAT. You can find upcoming dates here: http://www.manhattanlsat.com/zen-and-the-art.cfm
 
cyruswhittaker
Thanks Received: 107
Forum Guests
 
Posts: 246
Joined: August 11th, 2010
 
 
trophy
Most Thanked
trophy
First Responder
 

Re: PT 49, S 3, Q 21, "Which one of the following..."

by cyruswhittaker Mon Sep 27, 2010 7:37 pm

bbirdwell Wrote:
You're doing your job if you get it down to these two choices.


Yes but the problem is I keep getting it down to two choices...and then select the wrong one!! :roll:

But yes thank you for the explanation; it definately helps me to understand why E is not correct in this case.
 
adarsh.murthy
Thanks Received: 1
Jackie Chiles
Jackie Chiles
 
Posts: 32
Joined: November 03rd, 2011
 
 
 

Re: Q21

by adarsh.murthy Sun Nov 06, 2011 5:19 pm

I in fact also got it down to two choices and chose the right one :-). I was stuck between A and B. I eliminated B because of the word "many"? Do you also agree that "many" makes this option wrong?
wandering GMAT
 
mlbrandow
Thanks Received: 17
Jackie Chiles
Jackie Chiles
 
Posts: 29
Joined: January 22nd, 2012
 
 
 

Re: Q21

by mlbrandow Wed Sep 12, 2012 2:02 pm

I think even if we accept (wrongfully) that (E) has scientifically accurate information in it, and all of it is true about the passage, it misses the point.

The main point is the answer to the question at the end of the first paragraph--"But why are maize and a few other similar crops so much more bountiful than others?" (15-16)

Read those last few lines of the first paragraph, then reread (A).

Even if (E) were true (which it's not as bbirdwell stated), it would miss the point. It would still be lacking the context that (A) provides.

His point is that now we know why.
 
james.h.meyers
Thanks Received: 2
Vinny Gambini
Vinny Gambini
 
Posts: 19
Joined: June 07th, 2013
 
 
 

Re: Q21

by james.h.meyers Wed Sep 11, 2013 10:19 pm

adarsh.murthy Wrote:I in fact also got it down to two choices and chose the right one :-). I was stuck between A and B. I eliminated B because of the word "many"? Do you also agree that "many" makes this option wrong?


I don't think it's because of "many." On the lsat many just means some. I would say that it's because it misquotes the passage. I believe the passage does not state anything about the greater quantities of nutrients but rather the crops were successful because you could get greater quantities of the actual crop.

That said, I picked (A), and under timed conditions ended up crossing it out for (B). When I went back I realized my error.
User avatar
 
tommywallach
Thanks Received: 468
Atticus Finch
Atticus Finch
 
Posts: 1041
Joined: August 11th, 2009
 
 
 

Re: Q21

by tommywallach Mon Sep 16, 2013 6:31 pm

Hey James,

I agree. I don't see anything about nutrients in there. : )

-t
Tommy Wallach
Manhattan LSAT Instructor
twallach@manhattanprep.com
Image
 
olaizola.mariana
Thanks Received: 2
Elle Woods
Elle Woods
 
Posts: 52
Joined: May 12th, 2015
 
 
 

Re: Q21

by olaizola.mariana Tue Aug 25, 2015 10:32 am

Is it true that "On the lsat many just means some?" Are we not supposed to read "many" as implying "more than a few?" What, if anything, is the relevant difference between these two?
User avatar
 
tommywallach
Thanks Received: 468
Atticus Finch
Atticus Finch
 
Posts: 1041
Joined: August 11th, 2009
 
 
 

Re: Q21

by tommywallach Mon Aug 31, 2015 9:29 pm

No relevant difference. Many = some. Few = some. That's all.

-t
Tommy Wallach
Manhattan LSAT Instructor
twallach@manhattanprep.com
Image